Difference between revisions of "2016 AMC 8 Problems/Problem 16"

m (Solution)
Line 4: Line 4:
  
 
==Solution==
 
==Solution==
{{solution}}
 
 
Each lap Bonnie runs, Annie runs another quarter lap, so Bonnie will run four laps before she is overtaken.  That means Annie will have run <math>\boxed{5 }</math> laps.
 
Each lap Bonnie runs, Annie runs another quarter lap, so Bonnie will run four laps before she is overtaken.  That means Annie will have run <math>\boxed{5 }</math> laps.
 
{{AMC8 box|year=2016|num-b=15|num-a=17}}
 
{{AMC8 box|year=2016|num-b=15|num-a=17}}
 
{{MAA Notice}}
 
{{MAA Notice}}

Revision as of 12:33, 23 November 2016

Annie and Bonnie are running laps around a $400$-meter oval track. They started together, but Annie has pulled ahead, because she runs $25\%$ faster than Bonnie. How many laps will Annie have run when she first passes Bonnie?

$\textbf{(A) }1\dfrac{1}{4}\qquad\textbf{(B) }3\dfrac{1}{3}\qquad\textbf{(C) }4\qquad\textbf{(D) }5\qquad \textbf{(E) }25$

Solution

Each lap Bonnie runs, Annie runs another quarter lap, so Bonnie will run four laps before she is overtaken. That means Annie will have run $\boxed{5 }$ laps.

2016 AMC 8 (ProblemsAnswer KeyResources)
Preceded by
Problem 15
Followed by
Problem 17
1 2 3 4 5 6 7 8 9 10 11 12 13 14 15 16 17 18 19 20 21 22 23 24 25
All AJHSME/AMC 8 Problems and Solutions

The problems on this page are copyrighted by the Mathematical Association of America's American Mathematics Competitions. AMC logo.png